Header Ads Widget

Ticker

6/random

What is analytical grouping games? Explain with examples


Grouping Game

Grouping analytical games require the examinee to select a group of person, boy, girl, 

teacher, etc according to the set of conditions provide. 

Two examples of grouping analytical games are given below; 

Example No 3 

From time to time, the President of a company appoints planning Boards, each consisting of exactly 

three members. Eligible for appointment are three executives from Marketing—R, S, and T—and 

three executives from Finance—U, V, and W. Any given Board is subject to the following 

restrictions on appointments: 
  

At least one member must be from Marketing, and at least one member must be from Finance. 

If R is appointed, S cannot be appointed. 

Neither T nor V can be appointed unless the other is appointed also. 

If U is appointed, W must also be appointed. 


1. Which of the following is an acceptable Board?  

 (A) R, T, and W (B) S, V, and W (C) T. U, and V 

(D) T, V, and W (E) U, V, and W 


2. If appointees from Marketing are in the majority on a Board, that Board must include 

(A) R (B) S (C) U (D) V (E) W 


3.  If appointees from Finance are in the majority on a Board, that Board must include 

(A) R (B) S (C) U (D) V (E) W 


4.  If R is appointed to the same Board as W, which of the following will be true of that Board? 


(A)  Appointees from Marketing are in the majority. 

(B)  Appointees from Finance are in the majority.  

(C)  S is a Board member. 

(D)  V is a Board member. 

(E)   U is not a Board member. 

5.  If the restrictions on appointments apply also to a four-member Board appointed from the same group of executives, which of the following will be true? 

 (A)  If R is appointed, W must also be appointed. 

(B)   If S is appointed, U must also be appointed. 

(C)  If T is appointed, R must also be appointed. 

(D)  If V is appointed, S must also be appointed. 

(E)   If W is appointed, U must also be appointed. 


Solution 

Marketing Executive R S T 

Finance Executive U V W 

Exactly three-member board 

Rules 

Rule I, at least one member from Marketing and at least one member from Finance 

Rule II, R ≠ S 

Rule III, T = V 

Rule IV, U → W 

Question No 1 


1st Step Check Rule III A, B, E is wrong 

2nd Step Check Rule IV C is wrong 

So, Choice D is correct. 
  
Question No 2 
  
 Option 1 Option 2 



As for question and Rule II, then R S 

And remaining Marketing Executive, R T S T 

As Rule III, R T V S T V 

So, Choice D is correct. 


Question No 3 

 Option 1 Option 2 

As to the question and Rule IV, then U W V W 

As Rule III, T = V, so U W V W T 

Check the choices, 

Choice E is correct. 

Question No 4 

 Board 

As for a question, R W 

As Rule II, R ≠ S, so S cannot be appointed, 

As Rule III, T = V, so both T and V cannot be appointed 

Then remaining R W U 

So, Choice B is correct. 

Question No 5 

Take each choice and make the board according to the Rules 

Choice True/False Option 1 Option 2 

A True R 

 As Rule II, remaining R T 

 As Rule III, T = V, so R T V 

 As Rule IV, U cannot be 

 Inboard, so R T V W 

Choice A is the correct answer, as If R appoint then W must be appointed. 
  
Example No 4 

The editors of a journal that publishes three issues a year will devote the upcoming first, second, 

and third issues— in that order—exclusively to essays written by seven writers:   R, S, T, U, V, W, 

and X. Each of the seven writers will have at least one essay published, but some may have more than one essay published. The following restrictions apply to the publication of their essays: 

If an essay by R appears in an issue, then an essay by S must also appear in that issue.  

If an essay by U appears in an issue, then an essay by W must appear in the immediately preceding issue.  

An essay by W cannot be published in an issue that contains an essay by X.  

No writer may publish in each of two consecutively published issues or twice in the same issue.  

Each of the issues being prepared must contain at least two essays.  

The seven writers' essays can only appear in the upcoming first, second, and third issues. 



1. The first issue of the journal can consist exclusively of essays by which of the following groups of writers

(A) R and T 

(B) U and W 

(C) R, W., and X 

(D) T, V, and W 

(E) R, S, V, W, and X 

2. If the first issue consists exclusively of essays by R and S, then the second issue can consist exclusively of essays by which of the following groups of writers? 

(A) T and V 

(B) T and W 

(C) U and X 

(D) R, S, and X 

(E) T, W, and X 


3. Which of the following writers CANNOT contribute to the first issue of the journal? 

(A) T 

(B) U 

(C) V 

(D) W 

(E) X 

4. If the first issue consists exclusively of essays by S, T, and X, then the third issue must contain an 
 essay by which of the following writers? 

(A) S 

(B) T    

(C) V 

(D) W _ 

(E) X 

5. If the third issue consists exclusively of essays by S, T, and U, then the second issue must have con-
 sisted of essays by which of the following groups of writers? 

(A) R and V 

(B) R and X 

(C) S and W 

(D) V and W 

(E) V and X 


Solution 

 1 2 3 

First Issue Second Issue Third Issue 

Writer’s Essay R S T U V W X 

 At least one essay of one writer must be published 

Rules 

Rule I, R → S 

Rule II, W = U – 1 

Rule III, W ≠ X 

Rule IV, No writer may publish two essays in same Issue or Two consecutive Issue 

Rule V, Each issue must consist of at least two essays 

New Rule 

As Rule II & III, W = U – 1 and W ≠ X then X ≠ U – 1  

Question No 1 

1st Step Check Rule I A is wrong 

2nd Step Check Rule II B is wrong 

3rd Step Check Rule III C, E is wrong 

So, Choice D is correct. 

Question No 2 

 First Issue Second Issue Third Issue 

As for question, R S 

As Rule IV, R & S cannot publish 

In the Second Issue 

As Rule II, W = U – 1, then W must 

Publish in Second Issue, then R S W U 

As Rule III, W ≠ X 

Then remaining R S W T U 

Check the choices, Choice B is correct. 

Question No 3 

As Rule II, W = U – 1, then U ≠ 1 

So, Choice B is the correct answer. 

Question No 4 

 First Issue Second Issue Third Issue 

As for a question, S, T, X 

Then W must be published in Second 

An issue as Rule II, S, T, X W U 

R ≠ Second Issue, because as Rule I, 

R → S and S cannot publish in  

Second, the issues as Rule IV, then S, T, X W U, R, S 

Check the Choices, 

Choice A is correct. 

Question No 5 

 First Issue Second Issue Third Issue 

As for a question, S, T, U 

As Rule II, W = U – 1, so W S, T, U 

R ≠ Second Issue, because as Rule I, 

R → S and S cannot publish in  

Second Issue as Rule IV, then R, S W S, T, U 

As Rule III, W ≠ X, so X ≠ Second 

Issue 

Then remaining as IV, R, S W, V S, T, U 

Choice D is correct.